Maximum Sum Value

Given 10 non-negative reals such that a i i a_i \leq i and ( 1 a 1 ) ( 2 a 2 ) ( 9 a 9 ) ( 10 a 10 ) 1 (1-a_1)(2-a_2)\ldots(9-a_9)(10-a_{10}) \geq 1 , what is the maximum value of a 1 + a 2 + + a 9 + a 10 a_1 + a_2 + \cdots + a_9 + a_{10} ?


The answer is 45.

This section requires Javascript.
You are seeing this because something didn't load right. We suggest you, (a) try refreshing the page, (b) enabling javascript if it is disabled on your browser and, finally, (c) loading the non-javascript version of this page . We're sorry about the hassle.

12 solutions

Shyan Akmal
May 20, 2014

Let S = a 1 + a 2 + a 3 + . . . + a 10 S = a_1+a_2+a_3+...+a_{10} .

By AM-GM, we have k = 1 10 k a k 10 k = 1 10 ( k a k ) 10 = 1 \displaystyle \sum_{k=1}^{10}\frac{k-a_k}{10}\ge \sqrt[10]{\prod_{k=1}^{10}(k-a_k)} = 1\Rightarrow k = 1 10 ( k a k ) 10 \displaystyle \sum_{k=1}^{10}(k-a_k)\ge 10 \Rightarrow 10 11 2 S 10 \displaystyle \frac{10\cdot 11}{2} - S\ge 10 \Rightarrow 55 S 10 55 - S\ge 10 \Rightarrow S 45 S\le 45

It is easy to show that this bound is achievable. Indeed, AM-GM tells us that equality only holds if all the addends are equal. This means that 1 a 1 = 2 a 2 = 3 a 3 = . . . = 10 a 10 1-a_1=2-a_2=3-a_3=...=10-a_{10} However, the product of these numbers is 1 1 , so if all of them are equal, than each is equal to 1 1 . Setting each value equal to 1 1 , we find that a 1 = 0 a_1=0 a 2 = 1 a_2=1 a 3 = 2 a_3=2 . . . ... a 10 = 9 a_{10}=9 and this works, since S = 0 + 1 + 2 + . . . + 9 = 45. S=0+1+2+...+9=45. Thus, the maximum possible value of S S is 45 \boxed{45}

Common mistakes:

  1. You need to show that equality can be achieved. For example, it is also true that S 50 S \leq 50 , and this does not show that the maximum value is 50.
  2. It is not true that the maximum value of a i a_i is i 1 i-1 . For example, ( 10 ! 1 10 ! , 0 , 0 , 0 , 0 , 0 , 0 , 0 , 0 , 0 , 0 ) (\frac{ 10!-1} { 10!} ,0,0,0,0,0,0,0,0,0,0) is a valid solution to the inequality.

Calvin Lin Staff - 7 years ago

Log in to reply

The link to the blog provided in the problem doesn't work

Esp Ter - 4 years, 7 months ago

Log in to reply

Thanks. You can click on the "View wiki" link.

Calvin Lin Staff - 4 years, 7 months ago
Michael Tang
May 20, 2014

For brevity, we will denote the sum we want by S . S.

Consider the set A = { 1 a 1 , 2 a 2 , . . . , 10 a 10 } . A = \{1-a_1, 2-a_2, ..., 10 - a_{10} \}. Its arithmetic mean is

$$\frac{1}{10} (1-a 1+2-a 2+...+10-a {10}) $$ $$= \frac{1}{10}[55-(a 1+a 2+...+a {10})] = \frac{55-S}{10}$$

and its geometric mean is $$\sqrt[10]{(1-a 1)(2-a 2)...(10-a_{10})}.$$

We are given that ( 1 a 1 ) ( 2 a 2 ) . . . ( 10 a 10 ) 1 , (1-a_1)(2-a_2)...(10-a_{10}) \ge 1, thus ( 1 a 1 ) ( 2 a 2 ) . . . ( 10 a 10 ) 10 1 \sqrt[10]{(1-a_1)(2-a_2)...(10-a_{10})} \ge 1 as well.

We now use the AM-GM inequality:

55 S 10 ( 1 a 1 ) ( 2 a 2 ) . . . ( 10 a 10 ) 10 , \frac{55-S}{10} \ge \sqrt[10]{(1-a_1)(2-a_2)...(10-a_{10})}, so 55 S 10 1. \frac{55-S}{10} \ge 1. Solving this inequality for S S gives S 45. S \le 45. S = 45 S = 45 is indeed achievable (let a 1 = 0 , a 2 = 1 , . . . , a 10 = 9 a_1=0,a_2=1, ..., a_{10}=9 ), so the maximum possible value is 45 . \boxed{45}.

Hai Le
May 20, 2014

Using AM-GM inequality for 10 positive reals, we have \­(1\le (1-a 1)(2-a 2) \cdots (10-a {10}) \le {\left( {\frac{{1 - {a 1} + 2 - {a 2} + \cdots + 10 - {a {10}}}}{{10}}} \right)^{10}}.\­)

Therefore, a 1 + a 2 + + a 10 1 + 2 + + 9 = 45. {a_1} + {a_2} + \cdots + {a_{10}} \le 1 + 2 + \cdots + 9 = 45.

For a 1 = 0 , , a 10 = 9 a_1=0,\;\cdots,\;a_{10}=9 , we will have the equality.

Pranav Arora
May 20, 2014

From the AM-GM inequality,

( 1 a 1 ) + ( 2 a 2 ) . . . . . . . ( 9 a 9 ) + ( 10 a 10 ) 10 \frac{(1-a_1)+(2-a_2).......(9-a_9)+(10-a_{10})}{10} \geq ( ( 1 a 1 ) ( 2 a 2 ) . . . . . . . ( 9 a 9 ) ( 10 a 10 ) ) 1 / 10 1 ((1-a_1)(2-a_2).......(9-a_9)(10-a_{10}))^{1/10} \geq 1

55 i = 1 10 a i 10 1 \displaystyle \Rightarrow \frac{55-\sum_{i=1}^{10}a_i}{10} \geq 1

Rearranging and solving, i = 0 10 a i 45 \sum_{i=0}^{10} a_i \leq 45

Missing the equality condition

Calvin Lin Staff - 7 years ago
Jeffrey Robles
May 20, 2014

Let {(1-\a 1), (2-\a 2),...,(10-\a 10)} be the set of numbers of interest. Using the AM-GM inequality, we see that (i) [(1-\a 1)\cdot (2-\a 2)\cdot...(10-\a 10)]^1/10 \leq [(1-\a 1)+ (2-\a 2)+...+(10-\a_10)]/10.

The right-hand side of the inequality can be written as [55-(\a 1+\a 2+...+\a 10)]/10. Multiplying -1 to both side of inequality reverses the inequality, thus yielding (ii) -[(1-\a 1)\cdot (2-\a 2)\cdot...(10-\a 10)]^1/10 \geq [(\a 1+\a 2+...+\a_10)-55]/10.

But it is said that (1-a 1)(1-a 2)...(1-a_10) is greater than or equal to 1, then its negative is less than or equal to 1. Then the maximum value of the left hand side of inequality (ii) is -(1)^10. Rewriting (ii) yields,

(iii) -1^10 \geq [(\a 1+\a 2+...+\a_10)-55]/10.

With some manipulation, we see that (\a 1+\a 2+...+\a 10) \leq 45. Therefore, the maximum value of \a 1+\a 2+...+\a 9+\a_10 is 45 .

Need to show equality case

Calvin Lin Staff - 7 years ago
Advitiya Brijesh
May 20, 2014

As, the given equation is, ( 1 a 1 ) ( 2 a 2 ) ( 9 a 9 ) ( 10 a 1 0 ) 1 (1-a_1)(2-a_2)…(9-a_9)(10-a_10) \geq 1

Applying AM-GM inequality for LHS we get, ( 55 x 10 ) 10 1 (\frac{55-x}{10})^{10} \geq 1 (Where 'x' is the required sum)

It gives, x = 45 \boxed{x=45}

should write it out in pull properly, and state how it is used.

Calvin Lin Staff - 7 years ago
Sudipta Sarkar
May 20, 2014

{ (1−a1)+(2−a2)+…+(9−a9)+(10−a10) } / 10 > = {(1−a1)(2−a2)…(9−a9)(10−a10)}^1/10 => 55 - ( a1+a2+...+a10 ) >= 1 => a1+a2+...+a10 <= 45

Need to show equality case

Calvin Lin Staff - 7 years ago
Jiayang Zhao
May 20, 2014

Because a i i a_i \leq i , we know that each of 1 a 1 , 2 a 2 , . . . 10 a 10 1-a_1, 2-a_2,... 10-a_{10} are greater than or equal to 0. Since the product of 1 a 1 , 2 a 2 , . . . 10 a 10 1-a_1, 2-a_2,... 10-a_{10} is not 0, we know that none of 1 a 1 , 2 a 2 , . . . 10 a 10 1-a_1, 2-a_2,... 10-a_{10} can be equal to zero, thus they must all be simply greater than 0. Therefore, we can take the tenth root of both sides of the original inequality to get:

1 ( 1 a 1 ) ( 2 a 2 ) ( 3 a 3 ) . . . ( 10 a 10 ) 10 1 \leq \sqrt[10]{(1-a_1)(2-a_2)(3-a_3)...(10-a_{10})}

Note that the right side of the above equation is the geometric mean of 10 positive real numbers. By AM-GM, we know that the arithmetic mean of these 10 numbers is necessarily greater than their geometric mean. Therefore we have:

1 ( 1 a 1 ) ( 2 a 2 ) ( 3 a 3 ) . . . ( 10 a 10 ) 10 1 \leq \sqrt[10]{(1-a_1)(2-a_2)(3-a_3)...(10-a_{10})}

( 1 a 1 ) + ( 2 a 2 ) + ( 3 a 3 ) + . . . + ( 10 a 10 ) 10 \leq \frac{(1-a_1)+(2-a_2)+(3-a_3)+...+(10-a_{10})}{10}

= 1 + 2 + . . . + 10 ( a 1 + a 2 + . . . + a 10 ) 10 = \frac{1+2+...+10-(a_1+a_2+...+a_{10})}{10}

= 55 ( a 1 + a 2 + . . . + a 10 ) 10 = \frac{55-(a_1+a_2+...+a_{10})}{10}

Thus we have the inequality:

1 55 ( a 1 + a 2 + . . . + a 10 ) 10 1 \leq \frac{55-(a_1+a_2+...+a_{10})}{10}

Rearranging terms, we have the answer we are looking for:

a 1 + a 2 + . . . + a 10 45 a_1+a_2+...+a_{10} \leq 45

Need to show equality case

Calvin Lin Staff - 7 years ago
Calvin Lin Staff
May 13, 2014

Applying AM-GM to x i = i a i 0 x_i = i-a_i\geq 0 , we have ( 1 a 1 ) + ( 2 a 2 ) + + ( 9 a 9 ) + ( 10 a 10 ) 10 ( 1 a 1 ) ( 2 a 2 ) ( 9 a 9 ) ( 10 a 10 ) 10 1 \frac{(1-a_1)+(2-a_2)+\ldots+(9-a_9)+(10-a_{10})}{10} \geq \sqrt[10]{(1-a_1)(2-a_2)\ldots(9-a_9)(10-a_{10})} \geq 1 . Thus, 55 ( a 1 + a 2 + + a 9 + a 10 ) 10 55 - (a_1 + a_2 + \ldots + a_9 + a_{10}) \geq 10 a 1 + a 2 + + a 9 + a 10 45 \Rightarrow a_1 + a_2 + \ldots + a_9 + a_{10} \leq 45 . This can be achieved by setting a i = i 1 a_i = i-1 .

Curtis Clement
Feb 23, 2015

l e t A = a 1 + a 2 + . . . a 1 0 \large \ let \ A = a_1 +a_2 +...a_10 U s i n g t h e A M G M i n e q u a l i t y : \large \ Using \ the \ AM-GM \ inequality: ( 1 a 1 ) + ( 2 a 2 ) + . . . + ( 10 a 10 ) 10 [ ( 1 a 1 ) ( 2 a 2 ) ( 10 a 10 ) ] 1 10 = 1 \frac{(1-a_1) + (2-a_2) +...+(10-a_{10})}{10}\geq\ [(1-a_1)(2-a_2)\cdots\ (10-a_{10})]^{\frac{1}{10}} = 1 Now the maximum value of A is achieved when equality holds so multiplying by 10 and rearranging the terms: ( 1 + 2 + . . + 10 ) A = 10 55 A = 10 A = 45 (1+2+..+10) - A = 10 \Rightarrow\ 55 - A = 10 \therefore\ A = 45 e q u a l i t y h o l d s w h e n a i = i 1 \large \ equality \ holds \ when \ a_i = i-1

( n o t e : e q u a l i t y c a n h o l d i n o t h e r c a s e s \ note: \ equality \ can \ hold \ in \ other \ cases )

Here, the series is greater than or equal to 1

The maximum value of ai can be i

When ai=i then the given series becomes 0

So, the maximum possible value of ai will be 1 less than i

So the required maximum values of ai will be

a1=0 a2=1 a3=2 a4=3 a5=4 a6=5 a7=6 a8=7 a9=8 a10=9

The maximum value

a1+a2+a3+a4+a5+a6+a7+a8+a9+a10 = 0+1+2+3+4+5+6+7+8+9 = 45

                                      Answer: 45
Nolfe Violeta
May 20, 2014

(a 1,a 2,a 3,...,a 10) = (0,1,2,3, ...,9), max(sum) = (9 * 10) / 2 = 45

Not the only possibility

Calvin Lin Staff - 7 years ago

0 pending reports

×

Problem Loading...

Note Loading...

Set Loading...